2014 dxdy logo

Научный форум dxdy

Математика, Физика, Computer Science, Machine Learning, LaTeX, Механика и Техника, Химия,
Биология и Медицина, Экономика и Финансовая Математика, Гуманитарные науки


Правила форума


Посмотреть правила форума



Начать новую тему Ответить на тему На страницу Пред.  1, 2, 3, 4, 5, 6, 7  След.
 
 Re: Внутре гипотезы Коллатца
Сообщение20.11.2023, 21:21 
Заслуженный участник


20/08/14
11214
Россия, Москва
MGM в сообщении #1618955 писал(а):
$m_1$ (если имеется в виду последний шаг перед 1) всегда $2^{2n}$. Легко доказывается.
Это понятно: только такие степени двойки имеют остаток $1\bmod3$ и соответственно могут получиться предшествующей операцией $3x+1$.
Вопрос увидеть пример $m_1>4$ для минимальных $R_n$. До $10^{14}$ таковых не обнаружено. Не для минимальных - тривиально, можно построить например обратным ходом.
Из чисел вида $2^x-1$ минимальные $R_n$ дали только $x=2,3,5$ для $n=2,5,39$ (во всяком случае для $x<47$).

-- 20.11.2023, 21:30 --

MGM в сообщении #1618955 писал(а):
Однако, если считать все нечетные меньшие $2^{n}- 1$ подряд, [...]
Моего терпения хватило только до $n=30$. Дальше - часы ожидания.
Ну, смотря как и на чём писать программу. У меня в один поток считалось со скоростью около 140млрд/ч (т.е. около 7ч на триллионный интервал, а первый миллиард вообще секунд за 15). ;-)

 Профиль  
                  
 
 Re: Внутре гипотезы Коллатца
Сообщение20.11.2023, 21:38 
Аватара пользователя


07/01/16
1427
Аязьма
Dmitriy40 в сообщении #1618698 писал(а):
У меня две новости, и обе хорошие
И у меня кажется наконец хорошая новость, правда одна :-) Верю, что нашел разумный алгоритм расчета $R_n^{\min}$ (работающий быстрее сплошного перебора, начиная с достаточно больших $n$).
Смотрите:
1) $R_n^{\min}$ за один шаг приходит к какому-то числу вида $6k+1$ или $6k+5$; при этом, это наименьшие числа "класса" $n-1$ своего "рода" (т.е. дающие $1$ или $5$ по модулю $6$)
2) Эти два числа за шаг приходят к шести числам класса $n-2$, наименьшим в родах $1,5,7,11,13,17\bmod18$
...
n-1) Мы приходим к $2\cdot3^{n-1}$ числам первого класса, каждое из которых минимально в своем роду $\pmod{2\cdot3^{n}}$ (здесь все значения остатков, не делящиеся на три)

В таком виде задача, конечно, необозрима: огромное количество исходных кандидатов в первом классе, причем многие из них сами чудовищного размера. Здесь самое время вспомнить эвристику $s_n=\sum\ind_{i=1}^{n}{m_i}\approx\frac53n$. Это значит, слишком далеко бегать по получающемуся дереву не нужно: кандидатов первого класса может быть всего $\approx\frac23n$, а не $2\cdot3^{n-1}$. Это обозримо; к тому же, на каждом шаге будет фильтрация недопустимых значений по модулю $3$.

Проиллюстрирую на примере поиска $R_9^{\min}=43$, на котором обламывали зубья все мои более примитивные инструменты. Я немного сжульничаю и возьму $s_9=20>\frac53\cdot9=15$; это оправдано, для маленьких $n<50$ эвристика плохо работает; в общем, накинем немного, на фоне точного значения $2\cdot3^{8}=13122$ это ерунда. Пройдемся по дереву снизу вверх:

$m_1=4+2p$, при этом (по эвристике) должно выполняться $m_1+8\leqslant20$ ($8$ - это если остальные восемь $m_i$ принимают минимальное значение единица)- подходят только четные $4\leqslant{m_1}\leqslant12$; здесь я тоже сжульничаю и буду рассматривать только $m_1=4\Rightarrow{R_1=5}$ (это необязательно, просто в данном случае цепочки $m_i$ с $m_1>4$ заведомо не содержат решения и довольно быстро "загнутся"; в общем случае, для еще не найденных $R_n^{\min}$ так делать не следует; далее уже жульничать не будем); кстати же, $m_1=6$ мы бы и так отбросили, поскольку соответствующее ему $R_1=\frac13(2^6-1)=21$ делится на три и интереса не представляет;

$R_1=5\equiv2\pmod3$, и мы помним, что сумма всех $m_i$ не должна превзойти $20$, значит, далее возможны только такие цепочки для $R_2$ (Dmitriy40, простите, здесь все таки буду записывать $m_1$ слева, а $m_n$ справа, а то запутаюсь):$$\begin{array}{l}R_2=\\
(4,1)=3\text{ - неинтересно, т.к. делится на 3}\\
(4,3)=13\\
(4,5)=53\\
(4,7)=213\quad\text{- неинтересно, т.к. делится на 3}\\
(4,9)=853\end{array}$$
Аналогично, пририсовываем справа нечетное $m_3$ для $R_2\equiv2\pmod3$ и четное $m_3$ для $R_2\equiv1\pmod3$, не забывая, что в сумме все $m_i$ не должны превысить $20$, а так же выкидывая из дальнейшего рассмотрения получающиеся $R_3$, которые делятся на $3$:$$\begin{array}{l}R_3=\\
(4,3,2)=17\\
(4,3,6)=277\\
(4,5,1)=35\\
(4,5,5)=565\end{array}$$
$$\begin{array}{l}R_4=\\
(4,3,2,1)=11\\
(4,3,2,5)=181\\
(4,5,1,1)=23\\
(4,5,1,5)=373\end{array}$$
$$\begin{array}{l}R_5=\\
(4,3,2,1,1)=7\\
(4,3,2,1,3)=29\\
(4,3,2,5,2)=241\\
(4,5,1,1,3)=61\\
(4,5,1,1,5)=245\end{array}$$
$$\begin{array}{l}R_6=\\
(4,3,2,1,1,4)=37\\
(4,3,2,1,1,6)=149\\
(4,3,2,1,3,1)=19\\
(4,3,2,1,3,3)=77\\
(4,5,1,1,5,1)=163\end{array}$$
$$\begin{array}{l}R_7=\\
(4,3,2,1,1,4,2)=49\\
(4,3,2,1,3,1,2)=25\\
(4,3,2,1,3,1,4)=101\end{array}$$
$$\begin{array}{l}R_8=\\
(4,3,2,1,1,4,2,2)=65\\
(4,3,2,1,3,1,4,1)=67\end{array}$$
$$\begin{array}{l}R_9^{\min}=\\
(4,3,2,1,1,4,2,2,1)=43\end{array}$$Бинго! :-) Это я хочу (и кажется могу) запрогать.

 Профиль  
                  
 
 Re: Внутре гипотезы Коллатца
Сообщение20.11.2023, 22:27 
Заслуженный участник


20/08/14
11214
Россия, Москва
Да, я тоже думал над перебором $m_i$ в сторону увеличения, только собирался брать вектор сразу нужной длины и пытаться заполнять его с младших. Но до формализации ограничений руки так и не дошли.

 Профиль  
                  
 
 Re: Внутре гипотезы Коллатца
Сообщение20.11.2023, 23:53 
Аватара пользователя


07/01/16
1427
Аязьма
Dmitriy40 в сообщении #1618984 писал(а):
Да, я тоже думал над перебором $m_i$ в сторону увеличения, только собирался брать вектор сразу нужной длины и пытаться заполнять его с младших. Но до формализации ограничений руки так и не дошли.
Я в эту штуку прям верю - ну надо напрогать и проверить промышленно; сделаю не торопясь и криво. Конечно, она опирается на недоказанную эвристику, но зато наводит (если работает) какой-никакой порядок в бессистемных скачках "градин". Вообще, задача в такой постановке получается родственной с минимизацией сумм определенного вида и с разложением натурального числа в сумму заданного количества других натуральных, при определенных ограничениях. Так что не удивлюсь, если на эту тему есть известные результаты, я не пытался искать в литературе.

Еще пара соображений:
1) Алгоритм, запущенный для какого-то $n$, должен давать не только $R_n^{\min}$, но и все $R_k^{\min},k<n$

2) Я мог не жульничать, ведь эвристика выглядит как $s_n\approx\frac53n+7$, и можно было взять $s_9\leqslant22$; на результат бы не повлияло, только более честно показало бы объем вычислений. Ну, для иллюстрации, мне кажется, мелкое жульничество допустимо :-)

-- 21.11.2023, 00:00 --

MGM в сообщении #1618955 писал(а):
$m_1$ (если имеется в виду последний шаг перед 1) всегда $2^{2n}$. Легко доказывается.
Только это ничего не дает. Проблема глухая. Если идти путем алгебраических манипуляций.
Только все таки $m_1=4+2p,p\in\mathbb{N}_0$; у меня $m$ с каким-нибудь индексом - это степень двойки, на которую делится $3x+1$. Из всего, до чего дотянулись мои корявые ручки, только зависимость суммы этих $m$ от числа утроений выглядит гладко и ладно, я, соответственно, с этим фактом связываю надежду на (относительно) быстрое нахождение наименьших чисел, сваливающихся в единицу за $n$ утроений (обозначение $R_n^{\min}$ в тексте выше)

 Профиль  
                  
 
 Re: Внутре гипотезы Коллатца
Сообщение21.11.2023, 01:28 
Заслуженный участник


20/08/14
11214
Россия, Москва
waxtep
Мне кажется Вы для уменьшения перебора существенно опираетесь на недоказанную эвристику.

Я сделал перебор по своему, рекурсивно вычисляю $m_i$ в порядке увеличения $i$, с ограничением всех промежуточных $r_i$ на каждом шаге (без эвристик, максимально полно, но не больше текущего минимума), но оказалось совсем непрактично, он относительно быстро уменьшает $r_n$ до где-то стократной истинной величины и потом затыкается во внутренних переборах $m_i$. Так что дождаться скажем $n=60$ ещё можно, а вот длиннее ...
У Вас же не затыкание переборов будет, а разрастание с каждым шагом списка допустимых векторов, что только хуже (медленнее). И даже эвристика начнёт сильно ограничивать количество вариантов векторов лишь ближе к концу (это видно и из Вашего примера, количество стало уменьшаться лишь с седьмого шага из девяти).

waxtep в сообщении #1618991 писал(а):
1) Алгоритм, запущенный для какого-то $n$, должен давать не только $R_n^{\min}$, но и все $R_k^{\min},k<n$
Неа, он точно не найдет $R_n^{\min}=0\pmod3$, как например в Вашем примере не нашёл $R_6^{\min}=9$. Конечно это можно вернуть, тогда думаю да, найдёт и меньшие.

 Профиль  
                  
 
 Re: Внутре гипотезы Коллатца
Сообщение21.11.2023, 10:26 
Аватара пользователя


07/01/16
1427
Аязьма
Dmitriy40 в сообщении #1618999 писал(а):
Мне кажется Вы для уменьшения перебора существенно опираетесь на недоказанную эвристику.
Конечно, без этого вообще безумные объемы вычислений. Алгоритм выдает псевдо-минимальные числа, строго говоря.
Dmitriy40 в сообщении #1618999 писал(а):
У Вас же не затыкание переборов будет, а разрастание с каждым шагом списка допустимых векторов, что только хуже (медленнее)
Есть такой риск; там будет борьба разрастания вариантов с сокращением, за счет модулярных соображений и эвристического ограничения на $s_n$
Dmitriy40 в сообщении #1618999 писал(а):
Неа, он точно не найдет $R_n^{\min}=0\pmod3$, как например в Вашем примере не нашёл $R_6^{\min}=9$. Конечно это можно вернуть, тогда думаю да, найдёт и меньшие.
Найдет-найдет, я же иду снизу вверх, посмотрите на $R_2$. Я просто дальше такие не выписываю, поскольку ясно, что число более высокого класса не свалится в делящееся на три низкого класса.

 Профиль  
                  
 
 Re: Внутре гипотезы Коллатца
Сообщение21.11.2023, 11:25 
Заслуженный участник


20/08/14
11214
Россия, Москва
waxtep в сообщении #1619028 писал(а):
Я просто дальше такие не выписываю,
Ну я это и имел в виду, что если их не игнорить, то да, найдёт.

Но Вы понимаете что для меньших $n$ у Вас эвристика будет неправильной (слишком завышенной)? Она же для конечного $n$, а для меньших вариантов нужно меньше. Так что найдёт, но с огромным излишним (для меньших $n$) разрастанием списка векторов.

В общем способ (что Ваш, что мой) рабочий, но по моему всё же непрактичный. Разве что для $n>1000$ где-нибудь ... Куда линейным перебором за разумное время не добраться.

У меня счёт дошёл до 150e12, максимальное найденное $n=687$, минимальное не найденное $n=627$.

-- 21.11.2023, 12:00 --

waxtep в сообщении #1619028 писал(а):
Конечно, без этого вообще безумные объемы вычислений.
Ну, не всё так плохо: у меня почти моментально находится более-менее хорошее приближение сверху (вычисляю следующее $m_i$ как минимально допустимое при фиксированных предыдущих, для $n>70$ оно конечно улетает ввысь) и всё сваливается почти к той же эвристике (не точно, завышенно, зато без недоказанных предположений). Вот например динамика для $R_{40}=41$:
Код:
[4, 0, 0, 0, 0, 0, 0, 0, 0, 0, 0, 0, 0, 0, 0, 0, 0, 0, 0, 0, 0, 0, 0, 0, 0, 0, 0, 0, 0, 0, 0, 0, 0, 0, 0, 0, 0, 0, 0, 0]: Rn=10^99, s=4
[4, 3, 2, 1, 1, 4, 2, 2, 1, 4, 2, 1, 3, 2, 2, 4, 2, 1, 2, 4, 4, 2, 1, 4, 4, 2, 2, 1, 2, 3, 4, 4, 4, 2, 4, 4, 4, 2, 4, 2]: Rn=5354753308337, s=106
[4, 3, 2, 1, 1, 4, 2, 2, 1, 4, 2, 1, 3, 2, 2, 4, 2, 1, 2, 4, 4, 2, 1, 4, 4, 2, 2, 1, 2, 3, 4, 4, 4, 2, 4, 4, 4, 4, 1, 2]: Rn=2677376654169, s=105
[4, 3, 2, 1, 1, 4, 2, 2, 1, 4, 2, 1, 3, 2, 2, 4, 2, 1, 2, 4, 4, 2, 1, 4, 4, 2, 2, 1, 2, 3, 4, 4, 4, 2, 4, 6, 1, 2, 2, 1]: Rn=334672081771, s=102
[4, 3, 2, 1, 1, 4, 2, 2, 1, 4, 2, 1, 3, 2, 2, 4, 2, 1, 2, 4, 4, 2, 1, 4, 4, 2, 2, 1, 2, 3, 4, 4, 4, 2, 6, 3, 1, 1, 2, 1]: Rn=83668020443, s=100
[4, 3, 2, 1, 1, 4, 2, 2, 1, 4, 2, 1, 3, 2, 2, 4, 2, 1, 2, 4, 4, 2, 1, 4, 4, 2, 2, 1, 2, 3, 4, 4, 4, 4, 1, 2, 1, 5, 1, 2]: Rn=83668020441, s=100
[4, 3, 2, 1, 1, 4, 2, 2, 1, 4, 2, 1, 3, 2, 2, 4, 2, 1, 2, 4, 4, 2, 1, 4, 4, 2, 2, 1, 2, 3, 4, 4, 4, 4, 3, 1, 1, 1, 2, 1]: Rn=10458502555, s=97
[4, 3, 2, 1, 1, 4, 2, 2, 1, 4, 2, 1, 3, 2, 2, 4, 2, 1, 2, 4, 4, 2, 1, 4, 4, 2, 2, 1, 2, 3, 4, 6, 1, 2, 2, 1, 1, 4, 2, 1]: Rn=5229251275, s=96
[4, 3, 2, 1, 1, 4, 2, 2, 1, 4, 2, 1, 3, 2, 2, 4, 2, 1, 2, 4, 4, 2, 1, 4, 4, 2, 2, 1, 2, 3, 6, 3, 1, 1, 2, 5, 1, 1, 1, 2]: Rn=2614625641, s=95
[4, 3, 2, 1, 1, 4, 2, 2, 1, 4, 2, 1, 3, 2, 2, 4, 2, 1, 2, 4, 4, 2, 1, 4, 4, 2, 2, 1, 2, 5, 1, 2, 1, 3, 4, 2, 2, 2, 2, 1]: Rn=1307312811, s=94
[4, 3, 2, 1, 1, 4, 2, 2, 1, 4, 2, 1, 3, 2, 2, 4, 2, 1, 2, 4, 4, 2, 1, 4, 4, 2, 2, 1, 2, 5, 3, 1, 1, 1, 2, 3, 2, 2, 2, 1]: Rn=326828203, s=92
[4, 3, 2, 1, 1, 4, 2, 2, 1, 4, 2, 1, 3, 2, 2, 4, 2, 1, 2, 4, 4, 2, 1, 4, 4, 2, 2, 1, 6, 2, 3, 1, 1, 2, 1, 2, 2, 1, 1, 1]: Rn=81707055, s=90
[4, 3, 2, 1, 1, 4, 2, 2, 1, 4, 2, 1, 3, 2, 2, 4, 2, 1, 2, 4, 4, 2, 1, 4, 4, 2, 2, 3, 1, 1, 2, 1, 1, 4, 2, 1, 3, 1, 2, 2]: Rn=81707041, s=90
[4, 3, 2, 1, 1, 4, 2, 2, 1, 4, 2, 1, 3, 2, 2, 4, 2, 1, 2, 4, 4, 2, 1, 4, 4, 2, 6, 2, 2, 2, 1, 1, 2, 2, 1, 1, 1, 1, 2, 1]: Rn=40853531, s=89
[4, 3, 2, 1, 1, 4, 2, 2, 1, 4, 2, 1, 3, 2, 2, 4, 2, 1, 2, 4, 4, 2, 3, 1, 2, 3, 2, 1, 1, 3, 4, 2, 1, 1, 2, 2, 2, 2, 2, 2]: Rn=40853505, s=89
[4, 3, 2, 1, 1, 4, 2, 2, 1, 4, 2, 1, 3, 2, 2, 4, 2, 1, 2, 4, 4, 2, 3, 1, 2, 5, 1, 1, 1, 1, 2, 1, 2, 2, 2, 1, 1, 1, 1, 1]: Rn=319167, s=82
[4, 3, 2, 1, 1, 4, 2, 2, 1, 4, 2, 1, 3, 2, 2, 4, 2, 1, 4, 1, 2, 3, 1, 1, 2, 1, 2, 5, 2, 1, 2, 1, 1, 2, 2, 3, 2, 1, 1, 1]: Rn=319151, s=82
[4, 3, 2, 1, 1, 4, 2, 2, 1, 4, 2, 1, 3, 2, 2, 4, 2, 1, 4, 1, 2, 3, 1, 1, 2, 3, 1, 1, 1, 2, 1, 3, 4, 3, 1, 2, 1, 2, 1, 1]: Rn=319143, s=82
[4, 3, 2, 1, 1, 4, 2, 2, 1, 4, 2, 1, 3, 2, 2, 4, 2, 1, 4, 1, 2, 3, 1, 1, 2, 3, 1, 1, 3, 1, 1, 1, 2, 2, 1, 2, 1, 1, 1, 2]: Rn=19945, s=78
[4, 3, 2, 1, 1, 4, 2, 2, 1, 4, 2, 1, 3, 2, 2, 6, 1, 3, 2, 1, 2, 3, 1, 1, 1, 1, 1, 3, 1, 1, 1, 2, 1, 2, 1, 1, 1, 2, 1, 2]: Rn=4985, s=76
[4, 3, 2, 1, 3, 1, 4, 1, 2, 1, 3, 2, 1, 2, 5, 3, 3, 6, 1, 1, 2, 1, 1, 2, 2, 1, 1, 1, 3, 1, 1, 2, 1, 1, 1, 1, 1, 1, 1, 1]: Rn=2559, s=75
[4, 5, 1, 1, 3, 4, 2, 2, 4, 1, 1, 1, 1, 2, 1, 1, 1, 3, 4, 2, 1, 2, 1, 2, 2, 1, 1, 2, 1, 3, 1, 1, 1, 1, 4, 1, 2, 1, 1, 1]: Rn=1391, s=74
[4, 5, 1, 1, 3, 4, 2, 2, 4, 1, 1, 1, 3, 1, 1, 1, 1, 1, 2, 1, 2, 1, 1, 3, 2, 1, 1, 1, 2, 1, 1, 2, 1, 2, 2, 1, 1, 1, 1, 2]: Rn=41, s=69
n=40: Rn=41
Time: 1,653 ms
Но $R_{50}=731$ находится уже лишь за 44с. $R_{54}=1145$ за 99с. А $R_{60}=1583$ за 352с. Но если считать $n$ подряд, то для $R_{60}$ можно взять начальное ограничение не $10^{99}$, а $16R_{59}/3+1\approx6500$ (с запасом), в таком случае время будет 342с - 95% тратится на большие $m_i$ с малым $i$, для них ограничения оказываются слишком мягкими.
Верхний индекс указывать откровенно влом, ведь интересуют только с $\min$ и никакие другие.
Правда я тоже сжульничал: насильно установил $m_1=4$ и перебираю только следующие.

 Профиль  
                  
 
 Re: Внутре гипотезы Коллатца
Сообщение21.11.2023, 12:54 
Аватара пользователя


07/01/16
1427
Аязьма
Dmitriy40 в сообщении #1619035 писал(а):
, в таком случае время будет 342с - 95% тратится на большие $m_i$ с малым $i$, для них ограничения оказываются слишком мягкими.
Да, я подумал, что видимо еще жульничать придется - ограничивать не только сумму $m_i$, но и величину каждого $ m_i$ в отдельности. Помню, Вы в самой первой статистике анализировали, для какого $n$ впервые появляется следующее по величине $m_i$. Возможно, здесь тоже есть какая-то приближенная закономерность, имеет смысл посмотреть.

-- 21.11.2023, 12:55 --

Dmitriy40 в сообщении #1617685 писал(а):
4. До $n=503$ встретились варианты $m_i\in\{1,2,3,4,5,6,7,8,9,10,12,15\}$, первое $m_i=10$ для $R_{197}=1501353$, первое $m_i=12$ для $R_{348}=1224961663$, первое (и пока единственное) $m_i=15$ для $R_{502}=1210271371431$.
Вот это

 Профиль  
                  
 
 Re: Внутре гипотезы Коллатца
Сообщение21.11.2023, 13:20 
Заслуженный участник


20/08/14
11214
Россия, Москва
waxtep в сообщении #1619044 писал(а):
Да, я подумал, что видимо еще жульничать придется - ограничивать не только сумму $m_i$, но и величину каждого $ m_i$ в отдельности.
Так у меня практически так и делается: при обновлении рекорда $R_n$ я обновляю максимально допустимые промежуточные $r_{i<n}$, т.е. фактически $m_i$. Но чтобы не упираться в недоказанные эвристики беру $r_{i-1}=\lceil(3r_i+1)/2\rceil$ (реальное ограничение на $m_i$ при этом зависит от $m_{j<i}$ с меньшими индексами).
Дополнительное ручное ограничение $m_i<15$ эффекта не даёт: для $R_{54}=1145$ время уменьшается на жалкие 1.5с, меньше 2%.

-- 21.11.2023, 13:31 --

Вообще надо конечно поизучать в чём конкретно тормоза кода на PARI, а то окажется как обычно в самом PARI и переписывание на нормальном языке даст ускорение на пару-тройку порядков ...

 Профиль  
                  
 
 Re: Внутре гипотезы Коллатца
Сообщение21.11.2023, 18:50 
Аватара пользователя


05/06/08
474
waxtep в сообщении #1617495 писал(а):
Кажется, это что-то сезонное, по крайней мере я тоже маньячу в указанную гипотезу пару недель, независимо от появившихся недавно тем.
Не стал присоединяться к теме уважаемого Dmitriy40, поскольку интересуюсь частным вопросом: для данного натурального $n$ можно ли "легко" построить (хоть какое-то) натуральное нечетное $R_n$, которое "свалится в единицу" ровно за $n$ утроений? Далее попробую уточнить.


Задача тривиальная. Если нужно найти просто любое число с таким критерием (даже если остановить число умножений строго на первой единице). Таких чисел для любого $n $ бесконечное множество. Стройте обратное дерево от единицы. Более того, уже для $n=1$ чисел бесконечное множество.
$R_m^1 = \sum\limits_{i = 0}^m {{2^{2i}}} $
И каждые два числа из трех на уровне $R_m^n $ порождает бесконечное число чисел на уровне $ n+1$.
С минимальным числом - даже алгритм вряд ли сущесвует, не говоря уже об вменяемой аналитической формулы.
И все из-за тупиковых ветвей: если $R_m^n \equiv 0\bmod 3$ то это число - тупиковая ветвь.

PS Не совсем понятно, что вы хотите доказать в конце концов? Даже, допустим, если вы получите некоторою формулу типа:
$R_{\min }^n = f\left( n \right)$

 Профиль  
                  
 
 Re: Внутре гипотезы Коллатца
Сообщение21.11.2023, 19:22 
Заслуженный участник


20/08/14
11214
Россия, Москва
Ну, тут осталось два вопроса:
1. Все ли минимальные $R_n$ приходят к единице с последним шагом $5\to1$, т.е. имеют $m_1=4$ или есть и другие. Менее интересно есть другие варианты четвёрок $(m_4,m_3,m_2,m_1)$ кроме $(1,2,3,4),(1,1,5,4),(5,1,5,4)$. Я до 150e12 других не обнаружил.
2. Как вычислять $R_n^{\min}$ без полного перебора всех (или очень и очень многих, пока лишь с ограничениями $2R_n/3 < R_{n+1} < 16R_n/3+1$) нечётных. На данный момент шлифуем метод рекурсии с возвратом (я) и итерационного построения множества допустимых векторов $m_i$ на каждом шаге $j=1\ldots n$ (waxtep). По моему и тот и другой способы непрактичны, невзирая на все эвристики.

Процитированный вопрос, построение любого $R_n$, не обязательно минимального, несложен - поднятие по дереву с вычислением минимально подходящего $m_i$, но числа $R_n$ получаются на много (десятки!) порядков больше минимальных. Для них даже оценка приведена, по эвристике, но всё же.

 Профиль  
                  
 
 Re: Внутре гипотезы Коллатца
Сообщение21.11.2023, 19:52 
Аватара пользователя


05/06/08
474
Нет, конечно. 5ка, как порождающий элемент обратного дерева - конечно очень важная вершина, но вот уже число 85 сразу валится к единице. Бинарный делитель 256.

$85 \times 3 + 1 = 256 = {2^8}$
А первое число от 85ти на уровне выше вычисляется:
$\frac{{85 \times 4 - 1}}{3} = 113$
То есть 113 - это число в двух шагах от единицы и проходит не через 5, а через 85.
Но среди чисел второго уровня все же минимальное 3. И тройка сваливается к единице через 5ку.

 Профиль  
                  
 
 Re: Внутре гипотезы Коллатца
Сообщение21.11.2023, 19:53 
Аватара пользователя


07/01/16
1427
Аязьма
Сделал прототипчик, но да, считает очень долго, уже для $n=41$ у меня терпения не хватило. Отчасти, возможно, из-за возни с матрицами, но, факт, число вариантов растет слишком быстро. Надо еще думать, значит.
cz_eur(n)={my(sn=ceil(5*n/3)+7,px=floor((sn-n-3)/2)+1,Rcurr=matrix(px,3,p,col,(vectorv(px,v,1)*[(2^(4+2*(p-1))-1)/3,4+2*(p-1),4+2*(p-1)])[p,col]),Rnext=vector(4),R,s,m,ma,un);
for(i=2,n,
a=matsize(Rcurr)[1];
printp(vecmin(Rcurr[1..a,1]));
\\printp(Rcurr);
for(j=1,a,
R=Rcurr[j,1];
s=Rcurr[j,2];
m=Rcurr[j,3..i+1];
if(Mod(R,3)==1,px=floor((sn-s-n+i)/2);if(px>=1,ma=vectorv(px,k,2*k);un=vectorv(px,v,1);Rnext=matconcat([Rnext;matconcat([round((2^ma*R-un)/3),un*s+ma,un*m,ma])])));
if(Mod(R,3)==2,px=ceil((sn-s-n+i)/2);if(px>=1,ma=vectorv(px,k,2*k-1);un=vectorv(px,v,1);Rnext=matconcat([Rnext;matconcat([round((2^ma*R-un)/3),un*s+ma,un*m,ma])])));
);
Rcurr=Rnext[2..matsize(Rnext)[1],1..i+2];
Rnext=vector(i+3);
);
a=matsize(Rcurr)[1];
printp(vecmin(Rcurr[1..a,1]));
\\printp(Rcurr);
};


-- 21.11.2023, 19:55 --

MGM в сообщении #1619146 писал(а):
Но среди чисел второго уровня все же минимальное 3. И тройка сваливается к единице через 5ку.
В этом и вопрос: все ли минимальные сваливаются в единицу через пятерку?

 Профиль  
                  
 
 Re: Внутре гипотезы Коллатца
Сообщение21.11.2023, 21:00 
Аватара пользователя


05/06/08
474
waxtep в сообщении #1619147 писал(а):

MGM в сообщении #1619146 писал(а):
Но среди чисел второго уровня все же минимальное 3. И тройка сваливается к единице через 5ку.
В этом и вопрос: все ли минимальные сваливаются в единицу через пятерку?


Не думаю. Для достаточно больших чисел ($n=2^{1000}$, например ) скорее всего не так.
Но можете попробовать доказать следущее:
${m_1} = \mathop {\arg \min }\limits_{\left\{ {{m_1},{m_2},..{m_n}} \right\}} \left( {\frac{{{2^{{m_n}}} - {3^{n - 1}} - {3^{n - 2}}{2^{{m_1}}} - {3^{n - 3}}{2^{{m_2}}}... - {2^{{m_{n - 1}}}}}}{{{3^n}}}} \right) = 2$

где ${m_k} < {m_{k + 1}}$
Так как степень при двойке возрастает, а степень при тройке убывает, то не факт, что такое можно доказать.

 Профиль  
                  
 
 Re: Внутре гипотезы Коллатца
Сообщение21.11.2023, 21:03 
Заслуженный участник


20/08/14
11214
Россия, Москва
Моя программа, если захотите поиграться:
Код:
{Calc(n,i,rn)=my(x,y);\\rn - накопленное r по m[j<i]
   for(x=1,oo,\\цикл подбора m[i]
      y=rn*2^x-1;
      if(y>3*r[i], return);\\m[i] слишком велико
      if(y%3>0, next);\\m[i] не подходит, число не целое
      m[i]=x; y=y/3;
      if(i<n,
         if(y%3>0, Calc(n,i+1,y));\\внутри кратное трём недопустимо
      ,
         if(y>=r[n], next);\\m[n] слишком велико, нового меньшего Rn нет
         r[n]=y; forstep(y=n-1,1,-1, r[y]=ceil((3*r[y+1]+1)/2));\\пересчёт r[i] в обратном порядке от нового Rn=r[n]
         print(m,": R=",y,", s=",vecsum(m));\\показ процесса уменьшения Rn
      );
   );
}

n=40;
m=vector(n); r=vector(#m,i,3^n+100); m[1]=4;
print(m,": R=",r[n]);
Calc(n,2,(2^m[1]-1)/3);\\i=1 уже вычислено насильно
print("n=",n,": Rn=",r[n]);

 Профиль  
                  
Показать сообщения за:  Поле сортировки  
Начать новую тему Ответить на тему  [ Сообщений: 97 ]  На страницу Пред.  1, 2, 3, 4, 5, 6, 7  След.

Модераторы: Модераторы Математики, Супермодераторы



Кто сейчас на конференции

Сейчас этот форум просматривают: нет зарегистрированных пользователей


Вы не можете начинать темы
Вы не можете отвечать на сообщения
Вы не можете редактировать свои сообщения
Вы не можете удалять свои сообщения
Вы не можете добавлять вложения

Найти:
Powered by phpBB © 2000, 2002, 2005, 2007 phpBB Group